Sie sind auf Seite 1von 17

GRUPO QXMEDIC

Academia de Medicina

RESULTADOS DEL SIMULACRO

Fecha del Simulacro 13/02/2018 - 22:36

Nº de Preguntas 100

Nº de Preguntas Correctas 51

Nº de Preguntas Incorrectas 49

Calificación 10.2

Estado Finalizado

Pregunta 1 En relación con la etiología del RCIU asimetrico, señale factor más frecuente
CORRECTA
Enfermedad hipertensiva
Rubéola
Alcoholismo
Drogadicción
Tabaquismo

La respuesta correcta es: Enfermedad hipertensiva

Pregunta 2 Con respecto a la hiperémesis gravídica, indique que alternativa es la CORRECTA.


INCORRECTA
Existe relación con la infección por Helicobacter pylori
Se considera cuando baja más de 15% de peso
Es más frecuente en la segunda mitad del embarazo
Hay menor riesgo si es un embarazo gemelar
El apoyo psicológico no ayuda

La respuesta correcta es: Existe relación con la infección por Helicobacter pylori

Pregunta 3 Señale que cambio NO ocurre en el embarazo normal


CORRECTA
Albumina disminuye
Disminuye la tasa de flitracion glomerular
aumenta el reflujo vesicoureteral
4.Aumento del índice de flujo plasmático renal
elevación de factores de coagulación

La respuesta correcta es: Disminuye la tasa de flitracion glomerular

Pregunta 4 La maniobra utilizada en casos de inversión uterina:


INCORRECTA
Ritgen
Mac Roberts
Brand andrews
Jhonson
Bruch

La respuesta correcta es: Jhonson

Pregunta 5 ¿Cuál de las siguientes no se considera una complicación de las enfermedades de transmisión
CORRECTA sexual?

La neoplasia intraepitelial cervical


El mioma uterino
El embarazo ectópico
La infertilidad
La enfermedad inflamatoria pélvica

La respuesta correcta es: El mioma uterino

Pregunta 6 Hormona que se encuentra aumentada durante la gestación es:

1 / 17
GRUPO QXMEDIC
Academia de Medicina

INCORRECTA estrona
prolactina
prostaciclina
androstenediona
oxitocina

La respuesta correcta es: prolactina

Pregunta 7 ¿Qué decidua es la que se encuentra directamente por debajo del punto de implantación?
CORRECTA
Decidua capsular
decidua basal
decidua parietal
decidua lateral
decidua vellosa

La respuesta correcta es: decidua basal

Pregunta 8 No es una indicación de cesárea de urgencia:


CORRECTA
Desproporción céfalo pélvica
Placenta previa
Desprendimiento
RPM
RCIU severo

La respuesta correcta es: RPM

Pregunta 9 Medida ecográfica para predecir el riesgo de parto pretermino


CORRECTA
Longitud del cuello cervical
Longitud femur
Longitud DBP
Longitud espesor placentario
Longitud de OCI

La respuesta correcta es: Longitud del cuello cervical

Pregunta 10 En una mujer con 36 semanas de gestación y biopsia cervical que indica CÁNCER IN SITU, ¿cuál
INCORRECTA es la conducta obstétrica correcta?:

Continuar la gestación hasta la semana 38; luego indicar cesárea


Indicar cesárea de inmediato
De mejor pronóstico sería la cesárea histerectomía
Continuar la gestación hasta el término y permitir el parto vaginal
Realizar cono frío y luego procedes al parto vaginal

La respuesta correcta es: Continuar la gestación hasta el término y permitir el parto vaginal

Pregunta 11 El fármaco contraindicado en hemorragia post parto cuando existe preeclampsia es:
CORRECTA
Oxitocin,
Bmisoprostol,
Ergometrina
Relaxina
valsartan

La respuesta correcta es: Ergometrina

Pregunta 12 ¿Cuál es la masa ovárica más frecuente?


INCORRECTA
Cistoadenoma mucinoso benigno
Quiste endometroide benigno
Cistoadenofibroma
Tumor de Brenner
Adenofibroma

2 / 17
GRUPO QXMEDIC
Academia de Medicina

La respuesta correcta es: Quiste endometroide benigno

Pregunta 13 Cuando se expulsa la placenta, y vemos la cara fetal hablamos de un mecanismo de:
INCORRECTA
Shultza
Duncan
Jhonson
Roberts
Ritgen

La respuesta correcta es: Shultza

Pregunta 14 En el trabajo de parto normal, cuando la flexión se completa, el diámetro de la cabeza fetal que
INCORRECTA ingresa en el estrecho superior de la pelvis es

Occipitofronta
Suboccipitobregmático
A. Occipitomentoniano
A. Biparietal
A. Bitemporal

La respuesta correcta es: Suboccipitobregmático

Pregunta 15 Evalua el pronostico inicial de feto con RCIU


INCORRECTA
Doppler arteria umbilical
Doppler cordon umbilical
Doppler vena umbilical
Doppler de arteria uterina
Doppler arteria cerbral media

La respuesta correcta es: Doppler arteria umbilical

Pregunta 16 Al examen de una paciente se describen los siguientes datos en el perfil vaginal: cistocele de
CORRECTA tercer grado, uretrocele de segundo grado, no enterocele, histerocele de segundo grado, desgarro
perineovaginal de segundo grado y rectocele de segundo grado. En este caso, la fo?rmula del perfil
vaginal es:

2-2-0-2-3-2.
2-2-3-0-2-2.
2-3-2-0-2-2.
2-3-2-0-2-3.
3-2-0-2-2-2.

La respuesta correcta es: 2-3-2-0-2-2.

Pregunta 17 Paciente de 38 años, primigesta de 31 semanas con diagnóstico de ruptura prematura de


INCORRECTA membranas de 2 días de evolución, afebril y sin trabajo de parto. ¿Cuál de las siguientes
alternativas sirve para el diagnostico de corioamnionitis subclínica?

Disminución de los movimientos fetales


Glucosa menor de 15mg% en liquido amniótico
Perfil biofísico 6/8 por oligohidramios
Test no estresante no reactive
Hipersensibilidad uterine

La respuesta correcta es: Glucosa menor de 15mg% en liquido amniótico

Pregunta 18 OBSTETRICIA Gestante de 26 semanas de amenorrea que presenta desde hace 5 días malestar
CORRECTA general, astenia, náuseas, cefaleas, edemas y ligero dolor en hipocondrio derecho. En la analítica
practicada presenta:Hb 8 g/dl, bilirrubina 1,4 mg/dl, LDH 670 UI/I, plaquetas 80.00/mm3. Lo más
probable es que se trate de un embarazo complicado

Embolismo de líquido amniótico.


Muerte fetal con paso de sustancias tromboplasticas a la circulación materna.

3 / 17
GRUPO QXMEDIC
Academia de Medicina

Desprendimiento prematuro de placenta superior al 50%.


Isonmunizacion eritrocitaia con tranfusion feto-materna.
Preeclampsia severa.

La respuesta correcta es: Preeclampsia severa.

Pregunta 19 Primigesta de 19 años con 38 semanas de gestación. PA: 170/110 mmHg, proteinuria: 5g/L en
CORRECTA orina de 24 horas, plaquetas<100 000/mm3, deshidrogenasa láctica >600UI/L, hemolisis
microangiopatica. ¿Cuál es el diagnostico?:

Hipertensión crónica
Hipertensión crónica con síndrome de HELLP
Pre-eclampsia severa y alteración renal
Hipertensión gestacional con CID
Pre-eclampsia severa con síndrome de HELLP

La respuesta correcta es: Pre-eclampsia severa con síndrome de HELLP

Pregunta 20 Normalmente, en el embarazo se mantiene en rangos normales:


INCORRECTA
Proteínas totales
Estrona
Fosfatasa alcalina
Bilirrubina
Creatinina

La respuesta correcta es: Bilirrubina

Pregunta 21 GINECOLOGIA ¿Cuál sería el primer cambio hormonal de la pre-menopausia?


CORRECTA
FSH Y LH disminuidas
FSH Y LH aumentadas
FSH normal Y LH aumentadas
FSH aumentada y LH normal
FSH aumentada y LH disminuida

La respuesta correcta es: FSH aumentada y LH normal

Pregunta 22 ¿Cuál es el tumor de ovario que tiene origen histológico de células germinales y estromales
INCORRECTA respectivamente?

Mucinoso / granulosa
Seroso / tecoma
Teratoma / granuloso
Endometroide / teratoma
Teratoma / Cistoadenoma

La respuesta correcta es: Teratoma / granuloso

Pregunta 23 Gestante de 36 semanas de gestación a la que se diagnóstica hidrops fetal en ecografía de rutina.
INCORRECTA El COOMBs indirecto es positivo. El test de Kleihauer es negativo. El grupo y rH es 0 positivo. El
diagnóstico más probable es:

Isoinmunización rh.
Isoinmunización ABO.
Infección por parvovirus B19.
Transfusión feto-materna.
Idiopático.

La respuesta correcta es: Isoinmunización ABO.

Pregunta 24 ¿Cuál de los siguientes hallazgos colposcópicos es considerado normal?:


CORRECTA
Epitelio blanco
Mosaico
Leucoplaquia

4 / 17
GRUPO QXMEDIC
Academia de Medicina

Zona de transformación
Punteado hemorrágico

La respuesta correcta es: Zona de transformación

Pregunta 25 La indicación más frecuente de histerectomía es:


INCORRECTA
Hemorragia uterina anormal
Dolor pélvico crónico
Cáncer invasivo
Relajación pelviana
Leiomioma

La respuesta correcta es: Leiomioma

Pregunta 26 Mujer de 25 años acude a consulta con masa en mama de consistencia blanda, indolora, sin
INCORRECTA retracción cutánea y con algunas adenopatías axilares. Usted solicitaría una:

Mamografía y luego resonancia.


Mamografía y luego cirugía.
Ecografía y luego cirugía.
Ecografía y observación
Mamografía y observación.

La respuesta correcta es: Ecografía y observación

Pregunta 27 Gestante de 27 años de edad, con 30 semanas por ultima regla, G4, P2012, cesareada anterior 2
CORRECTA veces. Refiere sangrado en regular cantidad sin contracción . que solicito para confirmar
dagnostico:

TAC
RMN
Ecografia transvaginal
Radiografia pelvica
Cardiotocografia

La respuesta correcta es: Ecografia transvaginal

Pregunta 28 De los siguientes el factor más relacionado con la restricción crecimiento intrauterino tipo
CORRECTA simétrico es la:

Infección congénita
Isoinmunización Rh
Hipertensión
Dabetes con vasculopatía
Edad de la mujer por encima de 35 años

La respuesta correcta es: Infección congénita

Pregunta 29 ¿Cómo debe ser la atención prenatal?


CORRECTA
oportuna, periódica y completa
completa, regular y sofisticada
completa, oportuna y solidaria
regular, periódica y humanitaria
reenfocada solamente

La respuesta correcta es: oportuna, periódica y completa

Pregunta 30 ¿Cuál es el mecanismo que origina las desaceleraciones tempranas?


CORRECTA
Compresión del cordón umbilical
Oligohidramnios
Circular de cordón
Compresión de la cabeza fetal
Alteración de la reserva fetal de O2 fetal

5 / 17
GRUPO QXMEDIC
Academia de Medicina

La respuesta correcta es: Compresión de la cabeza fetal

Pregunta 31 A emergencia acude gestante en trabajo de parto, al realizar tacto vainal, se logra tactar el cordon
CORRECTA umbilical con presentación cefálica y membranas integras. El diagnostico correcto es:

Prolapso de cordon.
Procubito de cordon
Laterocidencia de cordon.
Procidencia de cordon.
Cordon velamentoso.

La respuesta correcta es: Procubito de cordon

Pregunta 32 Paciente de 40 años de edad, que acude a la consulta por sangrado postcoital de 2 semanas de
INCORRECTA evolución al examen pélvico:cérvix ulcerado hipertróficos, sangrante, poco móvil. Cuerpo uterino
6cm, anexos no palpables. Al tacto rectal: útero duro con parametrios tomados en sus 2/3 internos.
Ademas presenta hidronefrosis. El diagnóstico clínico es:

Cáncer de cérvix IIB


Cáncer de cérvix IIIA
Cáncer in situ
Cáncer de cérvix IIIB
Ectropión sangrante

La respuesta correcta es: Cáncer de cérvix IIIB

Pregunta 33 ¿Cuál de los siguientes tratamientos es el más adecuado en una mujer de 55 años, menopausica
CORRECTA desde hace uno por habérsele practicado una histerectomía con doble anexectomia, que padece
un síndrome climatérico importante?:

Calcio oral y benzodiacepina


Estrógenos y progestágenos 19 norderivados
Estrógenos y veralipride
Estrógenos y progesterona natural
Estrógenos exclusivamente

La respuesta correcta es: Estrógenos exclusivamente

Pregunta 34 Cuál es el principal criterio para considerar una HUA de tipo ovulatoria:
INCORRECTA
HIpermenorreica
Amenorreica
Hipomenorreica
Menorragica
Irregular .

La respuesta correcta es: Irregular .

Pregunta 35 ¿Cuál de los siguientes NO es factor de riesgo para cancer de utero?


CORRECTA
Síndrome de Lynch
Síndrome de Cowden
Terapia de reemplazo hormonal estrogenica
Terapia de reemplazo hormonal progestacional
Obesidad

La respuesta correcta es: Terapia de reemplazo hormonal progestacional

Pregunta 36 Mujer de 20 años, con cesárea por inducción fallida. Al tercer día de puerperio presenta malestar
CORRECTA general, fiebre y loquios fétidos. Al Examen clínico: T: 39°C, u?tero subinvolucionado y doloroso a
la palpacio?n, secrecio?n fe?tida. ¿Cuál es el tratamiento recomendable?

Ampicilina mas ceftazidima


Ampicilina mas gentamicina
Clindamicina mas amoxicilina

6 / 17
GRUPO QXMEDIC
Academia de Medicina

Clindamicina mas cefalexina


Ampicilina más eritromicina

La respuesta correcta es: Ampicilina mas gentamicina

Pregunta 37 Qué tipo de gemelos da origen la partición del disco embrionario a partir del 4 día de fecundación
INCORRECTA
Monoamniótico- bicoroiónico
Monoamniótico – Monocorionico
Biamniótico – Bicorionico
Biamniótico – Monocoriónico
Monstruos dobles

La respuesta correcta es: Biamniótico – Monocoriónico

Pregunta 38 ¿Cuál de las siguientes NO es un cambio fisiológico del embarazo?


INCORRECTA
Aumento de volumen plasmático
Aumento de tamaño de la hipófisis
Lordosis y flexión de miembros inferiores
Disminución de la viscosidad biliar
Aumento de la globulina transportadora de tiroxina.

La respuesta correcta es: Disminución de la viscosidad biliar

Pregunta 39 Valores séricos normales de hCG y su comportamiento durante el embarazo?


CORRECTA
Se triplican cada 72 horas
Se mantiene en meseta las primeras 10 semanas
A partir de que se detecta duplica su valor cada 2 a 3 días
Su pico maximo es en la 5ta semana
Aumenta durante toda la gestación

La respuesta correcta es: A partir de que se detecta duplica su valor cada 2 a 3 días

Pregunta 40 Paciente de 35 años, G: 6; P: 6-0-0-6; luego de un parto precipitado con recién nacido de 4,100 g
CORRECTA de peso y alumbramiento normal, presenta sangrado por vía vaginal, de color rojo rutilante y en
regular cantidad. Al examen: útero bien contraído por debajo de la cicatriz umbilical, doloroso; vulva
entreabierta, sin desgarros, donde se evidencia el sangrado descrito. La conducta terapéutica de
elección, luego de abrir la vía EV será:

Bolsa de hielo en el vientre


Taponamiento vaginal
Oxitócicos más ergometrina
Coagulantes
Ninguno de los mencionados

La respuesta correcta es: Oxitócicos más ergometrina

Pregunta 41 De los siguientes parámetros. ¿Cuál es el más importante en la evaluación clínica de la pelvis
INCORRECTA ósea en el estrecho medio?

Concavidad sacro coxígea


Diámetro biciatico
Diámetro bisquiáticas
Paredes pélvicas
Conjugado diagonal

La respuesta correcta es: Diámetro biciatico

Pregunta 42 Ante la sospecha de un embarazo tubárico no roto, cual es una indicación para tratamiento
CORRECTA conservador:

Paciente inestable
Liquido libre en cavidad
Saco gestacional > 4 cm

7 / 17
GRUPO QXMEDIC
Academia de Medicina

Ausencia de latidos cardiacos fetales


Localización en itsmo

La respuesta correcta es: Ausencia de latidos cardiacos fetales

Pregunta 43 Dosis estandar de Globulina inmune Rh D (anti D globulina inmune)?


INCORRECTA
300 microgramos
100 microgramos
500 microgramos
1000 microgramos
1 gr

La respuesta correcta es: 300 microgramos

Pregunta 44 ¿Cuál es la principal vía de diseminación del cáncer de ovario y de utero respectivamente?
INCORRECTA
Peritoneal / contiguedad de tejidos
Arterial / contiguedad de tejidos
Linfática / arterial
Peritoneal / linfatica
Transtubárica / arterial

La respuesta correcta es: Peritoneal / contiguedad de tejidos

Pregunta 45 ¿Cuál es el tipo de mioma más frecuente?


INCORRECTA
Submucosa
Subseroso
Mural
Abortado
Pediculado

La respuesta correcta es: Mural

Pregunta 46 Paciente de 26 años con 21 semanas de gestación, presenta dolor abdominal y sangrado vaginal
CORRECTA escaso, de 2 días de evolución. Al examen: altura uterina 20 cm, movimientos fetales presentes.
Especuloscopia se observa membranas ovulares prominentes e integras, que protruyen por el
orificio externo abierto. ¿Cuál es el diagnostico?

Óbito fetal
Aborto incompleto
Aborto inevitable
Aborto inminente
Aborto frustro

La respuesta correcta es: Aborto inminente

Pregunta 47 ¿Cuál de las siguientes es la INCORRECTA con respecto a los miomas uterinos?
INCORRECTA
La degeneración más frecuente es la grasosa
A la macroscopía tienen color blanco aperlado
Se inician a partir de un miocito del miometrio
Su crecimiento depende de hormonas
La degeneración roja es típica de embarazadas

La respuesta correcta es: La degeneración más frecuente es la grasosa

Pregunta 48 Paciente de 51 años, G:5; P:5004. Presenta tumoración que protruye por genitales. Al examen
INCORRECTA ginecológico se observa salida del cuello uterino a 6 centímetro de la línea himeneal (Punto
C:+6cm), tiene un TLV de 8. Según la clasificación de prolapso de órganos pélvicos (POP-Q) ¿Cuál
es el grado de histerocele?

IV
I
III

8 / 17
GRUPO QXMEDIC
Academia de Medicina

II
0

La respuesta correcta es: IV

Pregunta 49 Si un paciente tiene hemorragias irregulares asociado a infertilidad, tenemos una alta sospecha
INCORRECTA más probable de :

SOP
Endometriosis
Leiomioma
Cáncer de cuello
Cáncer de útero

La respuesta correcta es: SOP

Pregunta 50 Mujer de 35 an?os, que acude por tumoracio?n y dolor abdominal. Al Examen cli?nico:
INCORRECTA tumoracio?n anexial derecho de 7 cm, ascitis y derrame pleural. No signos de malignidad en
ima?genes, ni laboratorio. ¿Cua?l es el diagno?stico ma?s probable?

Quiste dermoide
Fibroma de ovario
Cistoadenoma seroso
Endometrioma
Cistoadenoma mucinoso

La respuesta correcta es: Fibroma de ovario

Pregunta 51 Multi?para con 35 semanas de gestacio?n, con control pre natal normal, y que despierta por la
INCORRECTA noche con pe?rdida sangui?nea vaginal mi?nima, sin dolor y ausencia de contracciones. ¿Cua?l es
la primera conducta a seguir?

Hospitalizacio?n y solicitar ecografi?a


Examen pe?lvico vaginal con guantes este?riles
Examen pe?lvico rectal con cuidado
Taponamiento vaginal y control del producto
Observacio?n en domicilio si reside cerca al Establecimiento

La respuesta correcta es: Hospitalizacio?n y solicitar ecografi?a

Pregunta 52 El útero bicorne es una anomalía congénita que se debe. Marque la respuesta correcta
INCORRECTA
Fusión anormal de los conductores de Muller o paramesonefricos
Fusión anormal de los conductos de Wolf
Alteración de la migración de las células primordiales
Alteración del desarrollo del tubérculo genital
Desarrollo incompleto del epooforon

La respuesta correcta es: Fusión anormal de los conductores de Muller o paramesonefricos

Pregunta 53 La diferenciación completa de los órganos genitales externos fetales se establecen desde la
CORRECTA
Octava semana
Décima semana
Vigésimo octava semana
Vigésima semana
Trigésimo segunda semana

La respuesta correcta es: Décima semana

Pregunta 54 Llega a consulta un paciente gestante de 30 semanas de gestación quien presenta cefalea y que
INCORRECTA en un control de una farmacia encontraron presión elevada en 2 oportunidades. Según su
sospecha diagnostica. ¿Marque la alternativa correcta?

La hipertensión en el embarazo es de la primera mitad del embarazo


El fármaco antihipertensivo inicial es nitroprusiato

9 / 17
GRUPO QXMEDIC
Academia de Medicina

Según la ACOG 2013 sigue siendo necesario tener proteinuria para diagnosticar de
preeclampsia
El tabaquismo es considerado un factor de riesgo
Se sugiere el uso de suplemento de calcio como prevecion de hipertensión en el embarazo

La respuesta correcta es: Se sugiere el uso de suplemento de calcio como prevecion de


hipertensión en el embarazo

Pregunta 55 Sólo una de las siguientes no es una manifestación clínica de la preeclampsia:


INCORRECTA
Proteinuria.
Excesivo aumento ponderal.
Edema maleolar
Presión diastólica mayor de 90 mmHg.
Cefalea.

La respuesta correcta es: Edema maleolar

Pregunta 56 En que momento se debe apli,car la gammaglobulina (vacuna Rh) Anti D?


INCORRECTA
Dentro de las primeras 72 hrs posparto.
30 ss
Hasta todo el puerperio
Solo puerperio inmediato
35 ss

La respuesta correcta es: Dentro de las primeras 72 hrs posparto.

Pregunta 57 Una mujer de 30 años, G5P5 Ao, acude a consulta por amenorrea. Refiere que en el último parto
CORRECTA presentó un cuadro de hemorragia postparto grave, con shock hipovolémico que requirio la
administración de 6 unidades de sangre. Lactancia artificial por no presentar secreción láctea.
Después del parto, la paciente ha tenido a parte de amonorrea, intolerancia al frío, fatiga, pérdida
de vello pubiano y axilar. ¿Usted que sospecharía ante éste cuadro? Señale la respuesta falsa:

Necrosis isquémica postparto.


Síndrome de Sheehan.
Síndrome de Asherman.
Se trata de una amenorrea que afecta al compartimento III de Speroff.
Existe un hipogonadismo-hipogonadotropo en presencia de otras deficiencias hipofisarias.

La respuesta correcta es: Síndrome de Sheehan.

Pregunta 58 ¿Cuál es el último movimiento cardinal durante el de parto?


INCORRECTA
Expulsión
Rotación externa
Rotación interna
Extensión
Flexión

La respuesta correcta es: Rotación externa

Pregunta 59 Durante el manejo expectante de una paciente gestante de 38 semanas que presento RPM hace
CORRECTA 12 horas y que ahora presenta dolor uterino, fiebre y leucocitosis con desviación izqueirda. ¿Cuál
es el siguiente paso en el manejo?

Cesarea programa al dia siguiente


Cesarea de emergencia
Antibióticos y esperar que mejore la infección
Culminar embarazo por la mejor via
Observación y reevaluar en 48 horas con nuevos controles

La respuesta correcta es: Culminar embarazo por la mejor via

Pregunta 60 Una de las siguientes presentaciones es indicación absoluta de parto mediante cesárea:
CORRECTA

10 / 17
GRUPO QXMEDIC
Academia de Medicina

Occipito transversa izquierda.


Occipito posterior.
Nalgas completas.
De cara mentoposterior.
De sincipucio

La respuesta correcta es: De cara mentoposterior.

Pregunta 61 El flujo sanguinolento por el pezón es más característico de:


CORRECTA
Ectasia ductal
Cáncer de mama
Papiloma intraductal
Mastalgia cíclica
Quiste de mama

La respuesta correcta es: Papiloma intraductal

Pregunta 62 GINECOLOGIA Mujer de 38 años, que presenta nódulo mamario indoloro, de bordes precisos.
CORRECTA Mamografía: imagen nodular con espículas en todos sus márgenes y finas micro calcificaciones
agrupadas en el interior. ¿Cuál es el diagnóstico más probable?

Fibroadenoma
Displasia mamaria
Quiste
Carcinoma
Papiloma intraductal

La respuesta correcta es: Carcinoma

Pregunta 63 ¿Cuál es el antibiótico de elección en pielonefritis aguda de una gestante cuyo resultado de
INCORRECTA Urocultivo mencionan E. coli BLEE positivo?

Meropenem
Ceftriaxona
Amoxicilina
Piperacilina mas tazobactam
Ampicilina mas sulbactam

La respuesta correcta es: Meropenem

Pregunta 64 Si una paciente presenta en una mamografía hecha de rutina o porque nota un bulto,
CORRECTA microcalcificaciones agrupadas, no diseminadas, el diagnostico de sospecha mas probable es:

Enfermedad fibroquistica
Necrosis grasa
Carcinoma
Galactocele
Mastitis crónica quistica

La respuesta correcta es: Carcinoma

Pregunta 65 Paciente de 36 años con HUA abundante diagnosticada de hiperplasia endometrial sin atipia.
INCORRECTA Acude por planificación familiar. Usted recomienda:

ACO
Solo progestágenos
DIU cobre
DIU levonorgestrel
Ligadura de trompas.

La respuesta correcta es: DIU levonorgestrel

Pregunta 66 GINECOLOGIA Mujer de 35 años de edad casada, que luego de legrado uterino hace 6 meses,
CORRECTA no presenta menstruación y sin actividad sexual hasta el momento de la consulta. Al examen; útero
y anexos normales. El medico solicita ecografía. ¿Cuál es el síndrome a tener en cuenta como

11 / 17
GRUPO QXMEDIC
Academia de Medicina

causa de amenorrea secundaria?

Asherman.
Ovario poliquístico.
Sheehan.
Kalman
Turner.

La respuesta correcta es: Asherman.

Pregunta 67 Con respecto a los DIU, marque la falsa:


INCORRECTA
En el MINSA brindan el DIU de cobre 400-A
La DIU hormonal dura 5 años
El DIU de cobre genera un proceso inflamatorio
Una contraindicación absoluta de colocar el DIU es el embarazo
El DIU hormonal no brinda el MINSA

La respuesta correcta es: En el MINSA brindan el DIU de cobre 400-A

Pregunta 68 En que momento se debe aplicar la gammaglobulina (vacuna Rh) Anti D?


CORRECTA
Entre 20 y 24 semanas
Solo en las 72 horas posparto
28 semanas y dentro de las primeras 72 hrs posparto.
Solo a las 28 semanas si el RN es rh positivo
A las 32 semanas

La respuesta correcta es: 28 semanas y dentro de las primeras 72 hrs posparto.

Pregunta 69 ¿Cuál es la principal función protectora que ejercen las contracciones uterinas después del
CORRECTA alumbramiento?

analgesia
inmunologica
antihipertensiva
La hemostasia
Antinauseosa

La respuesta correcta es: La hemostasia

Pregunta 70 Paciente de 50 años con hemorragia uterina anormal y biopsia uterina informada como
CORRECTA hiperplasia simple atípica pero la señora no desea ninguna intervención quirúrgica. Cuál es la
conducta a seguir:

Tratamiento con acetato de medroxiprogesterona


Histeroscopia
Histerectomía abdominal total si o si
Esperar un nuevo control
Ablación endometrial

La respuesta correcta es: Tratamiento con acetato de medroxiprogesterona

Pregunta 71 Mujer de 43 años, remitida a la Consulta de Patología del Tracto Genital Inferior del Hospital por
INCORRECTA presentar una citología cérvico-vaginal informada como “L-SIL” (Lesión Intraepitelial de Bajo
Grado). Refiere infecciones vaginales de repetición (Trichomoniasis,…) y haber tenido más de 5
parejas sexuales en su vida. La mejor estrategia de atención a esta mujer es:

Conización con asa de diatermia


Legrado endocervical con legra de Kevorkian
Biopsia de endometrio con cánula de Cornier.
Histerectomía sin anexectomía.
Aconsejarle que deje el tabaco y los anticonceptivos y hacerle una colpoccopia

La respuesta correcta es: Aconsejarle que deje el tabaco y los anticonceptivos y hacerle una

12 / 17
GRUPO QXMEDIC
Academia de Medicina

colpoccopia

Pregunta 72 Paciente de 22 an?os FUR hace 6 di?as, llega a emergencia presentando dolor en hipogastrio y
INCORRECTA fiebre, al tacto vaginal se encuentra tumoracio?n anexial bilateral. Posible diagno?stico:

absceso tubo-ova?rico
endometritis aguda
tuberculosis genital
salpingitis aguda
sindrome adherencial

La respuesta correcta es: absceso tubo-ova?rico

Pregunta 73 Una mujer de 44 años de edad, con antecedente de dos embarazos normales y ningún dato
CORRECTA patológico destacable en su historia ginecológica. Acide a consulta para revisión refiriendo que ha
sido controlada en medicina interna para descartar un cáncer de colon no poliposico hereditario
(síndrome de Lynch tipo II). Esta paciente tiene un elevado riesgo de padecer un cáncer de colon a
lo largo de su vida, pero también de padecer otros tipos de neoplasias. Señale cuál de los
siguientes canceres ginecológicos es el que implica un mayor riesgo de presentación en esta
paciente:

Cáncer de mama
Cáncer de vulva
Cáncer de ovario
Cáncer de endometrio
Cáncer de cuello uterino

La respuesta correcta es: Cáncer de endometrio

Pregunta 74 Una mujer de 42 años con implantes mamarios consulta por tumor palpable en cuadrantes
INCORRECTA internos de mama derecha. ¿Qué prueba debe indicarse para descartar un cáncer?

Ecografía mamaria
Mamografía
Resonancia magnética
TAC
PET

La respuesta correcta es: Resonancia magnética

Pregunta 75 Manifestación clínica más frecuente de embarazo molar después de la metrorragia?


INCORRECTA
preeclampsia
aborto
hipertiroidimos
quistes tecagranulosos
hiperemesis

La respuesta correcta es: hiperemesis

Pregunta 76 ¿Cuál es cierto con respecto a hiperémesis gravídica?


CORRECTA
Se presenta acidosis metabolica
La teoría nutricional es la mas aceptada
El antiemético inicial es diminhidrinato
La complicación más severa se produce por déficit de tiamina
Debemos evitar un próximo embarazo

La respuesta correcta es: La complicación más severa se produce por déficit de tiamina

Pregunta 77 ¿Cómo se define RCIU severo?


INCORRECTA
< p5
< p2
< p10

13 / 17
GRUPO QXMEDIC
Academia de Medicina

< p1
< p4

La respuesta correcta es: < p2

Pregunta 78 ¿Cuál de las siguientes no es una manifestación clínica característica de la endometriosis?


CORRECTA
Hipermenorrea
Dismenorrea
Esterilidad primaria
Dolor pélvico crónico
Dispareunia

La respuesta correcta es: Hipermenorrea

Pregunta 79 OBSTETRICIA Primigesta de 32 años de edad, con 36 semanas de gestación y control prenatal
CORRECTA irregular. Presenta desde hace 48 horas cefalea, visión borrosa y dolor epigástrico.PA:180/120
mmHG, FC:110 por minuto, ictericia leve. Leucocitos: 12.000/mm3, hemoglobina:8g%,
plaquetqas:30.000, TGP:300, bilirrubina total: 3; bilirrubina indirecta:2,5: creatina:1,2 examen de
orina: proteínas(+++), hematuria. La medida más adecuada será:

Transfundir plaquetas y esperar resultados.


Admisión en UCI e inicio de hipotensores endovenosos.
Terminar la gestación de inmediato.
Iniciar hipotensores agresivamente y esperar resultados.
Interconsulta a nefrología y tto antihipertensivo.

La respuesta correcta es: Terminar la gestación de inmediato.

Pregunta 80 Gestante de 37 semanas de edad gestacional que presenta PA 140/90, proteína en orina de 24
INCORRECTA horas 350mg/dl niega otras alteraciones y bioquímica solicitada normal la conducta en este casi
seria:

Control ambulatorio y esperar que se inicie el parto espontaneo.


Programar para cirugía cesárea
Uso de antihipertensivos y esperar el único espontaneo del parto
Maduración – Inducción
Ninguna de las anteriores

La respuesta correcta es: Maduración – Inducción

Pregunta 81 ¿Qué fármaco está contraindicada en gestantes?


INCORRECTA
cloroquina
heparina
insulina
leflunamida
propiltiouracilo.

La respuesta correcta es: leflunamida

Pregunta 82 Gestante de 35 semanas, nulípara, presenta hace 6 horas dolor abdominal. Al examen físico:
CORRECTA dinámica uterina de intensidad (++), de 30 segundos de duración, con frecuencia de 3 minutos. Al
tacto vaginal: dilatación de 3 cm y borramiento 80 %. ¿Cuál es el diagnóstico más probable?

Contracciones de Braxon
Amenaza de parto prematuro
Irritabilidad uterina
Rotura uterina
Trabajo de parto prematuro

La respuesta correcta es: Trabajo de parto prematuro

Pregunta 83 Gestante de 18 semanas, acude a Emergencia con dolor abdominal pélvico tipo contracción, con
CORRECTA escaso sangrado vaginal, afebril. Al examen ginecológico se aprecia perdida de líquido anmiótico y
cuello dilatado. El diagnóstico probable es:

14 / 17
GRUPO QXMEDIC
Academia de Medicina

Aborto inevitable
Aborto séptico
Aborto retenido
Aborto incompleto
Aborto completo

La respuesta correcta es: Aborto inevitable

Pregunta 84 En la presentacio?n cefa?lica de frente, cua?l es el punto de referencia de la cabeza fetal:


CORRECTA
Mento?n
Glabela
Fontanela anterior
Bregma
Ocipucio

La respuesta correcta es: Glabela

Pregunta 85 La complicación más temida por el abruptio placenta es:


CORRECTA
Déficit de vitamina K
Hipofobrinogemia
Púrpura trombocitopénica
Déficit de gammaglobulina
Ninguna de las anteriores

La respuesta correcta es: Hipofobrinogemia

Pregunta 86 En relación a la enfermedad hemolítica por isosensibilizacion ABO, señale la FALSA:


INCORRECTA
La madre suele ser O.
Los anticuerpos causantes suelen ser los de la clase lg G.
Pueden resultar afectos los productos del primer embarazo
En antígeno más frecuentemente implicado es A1.
La enfermedad se agrava en los embarazos siguientes, sino hacemos profilaxis.

La respuesta correcta es: La enfermedad se agrava en los embarazos siguientes, sino hacemos
profilaxis.

Pregunta 87 Gestante de 36 años, G:4, P:003, 20 semanas por última regla y altura uterina, PAP: lesión
INCORRECTA intraepitelial de alto grado, la coloscopía satisfactoria muestra epitelio acetoblanco y mosaico en el
radio de las 12. ¿Cuál es el paso más apropiado a seguir?:

Biopsia dirigida
Cesárea- histerectomía a las 38 semanas
Conización con asa de alambre
Electrocauterización
Repetir PAP y colposcopía 45 días después del parto

La respuesta correcta es: Biopsia dirigida

Pregunta 88 Durante el embarazo ocurren eventos fisiológicos, ¿Cuál de los siguientes no es uno de ellos?
CORRECTA
Edema maleolar
Caminata de orgullo
Disminución del número de eritrocitos
Aumento de casi todos los factores de la coagulación
Aumento del fibrinógeno

La respuesta correcta es: Disminución del número de eritrocitos

Pregunta 89 Mujer de 30 an?os consulta por dolor en las mamas cada vez que menstrua. Al Examen fi?sico:
INCORRECTA no tumoraciones, ecografi?a de mama: normal. Cua?l es el diagno?stico ma?s probable?

Mastodinia ciclica
Mastalgia ci?clica

15 / 17
GRUPO QXMEDIC
Academia de Medicina

Fibroadenoma de mama
Mastopatia fibroquistica
Si?ndrome premenstrual

La respuesta correcta es: Mastalgia ci?clica

Pregunta 90 Paciente de 55 años que presenta sangrado post coital, flujo genital mal oliente, Papanicolaou:
CORRECTA LIE de alto grado. Al examen: lesión exofitica de 3 cm en el cuello uterino, sin compromiso de
parametrios. Según la clasificación FIGO para cáncer cervical. ¿A qué etapa corresponde?

I (Estadio IB1: Lesiones clínicas de tamaño máximo de 4 cm)


II
0
IV
III

La respuesta correcta es: I (Estadio IB1: Lesiones clínicas de tamaño máximo de 4 cm)

Pregunta 91 OBSTETRICIA ¿Cuál es el enunciado correcto en relación al crecimiento fetal:


CORRECTA
El tipo más frecuente de RCIU es el simétrico.
El RCIU tipo I tiene como causa insuficiencia placentaria.
El crecimiento en talla es tardío posterior al peso
El RICU tipo II es de mejor pronostico
Los niños por debajo del percentil 5 presentan RCIU.

La respuesta correcta es: El RICU tipo II es de mejor pronostico

Pregunta 92 . Mujer de 16 años que consulta por amenorrea primaria. Presenta un desarrollo femenino normal
CORRECTA con caracteres sexuales secundarios. Los niveles de estradiol y testosterona son normales. En la
exploración ginecológica se aprecia agenesia de vagina. Se realiza ecografía y se aprecia ausencia
de útero. Los ovarios son normales ecográficamente. No se observa riñón izquierdo. El diagnóstico
más probable es:

Síndrome de Rokitanski.
Síndrome de Morris.
Hiperplasia suprarrenal congénita.
Síndrome de ovarios poliquístico
Síndrome de Kallman

La respuesta correcta es: Síndrome de Rokitanski.

Pregunta 93 Gestante de 22 an?os, con 18 semanas de gestacio?n por FUR, grupo sangui?neo A, factor Rh
INCORRECTA negativo. Antecedente de un parto euto?cico, donde recibio? inmunoglobulina Anti D. El embarazo
actual es de su segundo compromiso. ¿Cua?l es la conducta adecuada?:

Esperar porque no hay problema.


Inmunoglobulina anti D a las 26 y 32 semanas.
Inmunoglobulina Anti D dosis u?nica.
Test de Liley.
Tipificacio?n sangui?nea de la pareja.

La respuesta correcta es: Tipificacio?n sangui?nea de la pareja.

Pregunta 94 En el manejo de una gestante de embarazo de 32 semanas que inicia trabajo de parto y que al
CORRECTA momento tiene evidencia de inmadurez pulmonar. ¿Qué indicaría?

Terminar la gestación por cesárea


Terminar la gestación por parto vaginal
Iniciar antibióticos como tratamiento
Indicar corticoides
Suspender tocoliticos

La respuesta correcta es: Indicar corticoides

Pregunta 95 Gestante de 39 semanas en fase activa de trabajo de parto. Feto en presentacio?n cefa?lica con

16 / 17
GRUPO QXMEDIC
Academia de Medicina

CORRECTA deflexio?n y punto de referencia nasal. ¿Cua?ntos centi?metros mide el dia?metro que se
presenta?

11
12.5
13
13.5
14

La respuesta correcta es: 13.5

Pregunta 96 La presentación fetal más frecuente en el embarazo gemelar que genera distocia en el momento
INCORRECTA del parto es:

Pelviana - cefalica
Cefálica - pelviana.
Cefálica - cefálica.
Transversa - transversa.
Pelviana – transversa

La respuesta correcta es: Pelviana - cefalica

Pregunta 97 El aborto recurrente o habitual se define como la pérdida gestacional repetida en un MÍNIMO de:
CORRECTA
5
3
2
6
4

La respuesta correcta es: 3

Pregunta 98 ¿Cuál de las siguientes patologías acelera la maduración pulmonar fetal?


INCORRECTA
Diabetes mellitus
Preeclampsia
Isoinmunizacion
Anemia severa
Hiperémesis gravídica

La respuesta correcta es: Preeclampsia

Pregunta 99 El corticoide prenatal para madurar el pulmón que se indica es ………..; durante la edad gestacional
CORRECTA de…………:

Betametasona 12 mg c/12 hrs Entre 20 y 24 semanas


Dexametasona 6 mg c/24 hrs Entre 24 y 33 semanas
Betametasona 12 mg c/24 hrs Entre 24 y 33 semanas
Dexametasona 6 mg c/24 hrs Entre 30 y 34 semanas
Betametasona 12 mg c/12 hrs Entre 20 y 24 semanas

La respuesta correcta es: Betametasona 12 mg c/24 hrs Entre 24 y 33 semanas

Pregunta 100 Con respecto al Test estresante es cierto excepto:


INCORRECTA
Se provoca contracciones con la administración de oxitocina.
Relaciona actividad cardiaca fetal con las contracciones uterinas
Evalúa la reservación placentaria
Predice afectaciones agudas fetales como accidentes del cordón
Se dice que es positiva si hay desaceleraciones tardías el 40% o más del trazado.

La respuesta correcta es: Se dice que es positiva si hay desaceleraciones tardías el 40% o más
del trazado.

Si tiene dudas de alguna pregunta por favor escribir al siguiente email christiam.ochoa@qxmedic.com

17 / 17

Powered by TCPDF (www.tcpdf.org)

Das könnte Ihnen auch gefallen